2

From the on-shell action, we derive the following two:

  • $\frac{\partial S}{\partial t_1} = H(t_1)$,
  • $\frac{\partial S}{\partial t_2} = -H(t_2)$,

where $H = vp - L$ is the energy function.

I have two following questions mostly related to each other.

Q1: we ask how much S changes when we change $t_1$. This basically means we're not changing the path in variation, but we're just saying what if we had started the path from $t1+\delta h$ which means the remaining path is the same. Correct?

Q2: I wonder how we derived $\frac{\partial S}{\partial t_1} = H(t_1)$. My idea is as follows - we know that $H = vp - L$ from which we have: $L = vp - H$.

I also understand that $\frac{\partial S}{\partial t_1} = -L(t1)$.

So we have:

$\frac{\partial S}{\partial t_1} = -(vp - H)(t1)$

It seems like $vp$ is considered as 0 here. Is this because at time $t1$, the speed is 0 ? but if we're observing the motion from $t=2$ to $t=5$, at $t=2$, speed wouldn't be 0. So why and how did we make it 0?

You can follow the answer how these equations I wrote are derived on the following answer.

Qmechanic
  • 201,751
Giorgi
  • 525
  • The energy (hamiltonian) at the final point $t_2$ is equal to $-\frac{\partial S_{cl}}{\partial t_2}$ and the energy at the initial point is equal to $+\frac{\partial S_{cl}}{\partial t_1}$. This is kind of an obscure notion, but it is nevertheless true. You can see this is true from the very simple example of the classical action for a free particle: $S^{\text{free}}_{cl} = \frac{m}{2}\frac{(x_2 - x_1)^2}{(t_2 - t_1)}$. – hft Jul 21 '23 at 22:50
  • @Chemistry This point is a little hard to explain without drawing a picture. But, basically, you consider the $t_1$ endpoint and when you displace that forward in time, you move off of the original classical path. But you move on to a different classical path, so you get contributions from the integrand (as you would expect), but you also get a contribution from the fact that you are on a different path (this is the part that is hard to explain without a picture). – hft Jul 21 '23 at 22:56
  • @hft lets say t is x axis, and x is y axis. When you change t1 to ta(i call it ta to distinguish) that ta point is next to t to the right and we consider it as starting point. At t=ta, you could choose infinite amounts of x as starting points, but for sure one of them from these infinite points is on our original classical path. Are we not choosing the one on our original path but what ? Is this not anywhere explained really ? – Giorgi Jul 21 '23 at 23:09
  • @Chemistry When you write: "I also understand that $\frac{\partial S}{\partial t_1} = -L(t1)$." This is only correct if you do not modify the classical path. But the tricky part is that you do have to modify the classical path. You can not just use the usual rule of differentiation with respect to the endpoint of an integral. If you just use the usual rule and dont modify the path you get the wrong answer. – hft Jul 21 '23 at 23:11
  • @Chemistry In the meantime take a look at the solution to problem 2-5 here: https://www.gotohaggstrom.com/Solutions%20to%20Feynman-Hibbs%20classical%20action%20problems.pdf https://www.amazon.com/Quantum-Mechanics-Path-Integrals-Emended/dp/0486477223 – hft Jul 21 '23 at 23:16
  • This is a duplicate of eq. (12) in my Phys.SE answer here where a proof is given. – Qmechanic Jul 22 '23 at 10:50

1 Answers1

3

I also understand that $\frac{\partial S}{\partial t_1} = -L(t_1)$.

This is only true when the integrand is fixed. When the integrand is not fixed, we can not use the usual rule for differentiation with respect to a lower limit of an integral.

This is why $$ \frac{\partial S}{\partial t_1} = - L(t_1)\tag{$L(x(t),\dot x(t))$ fixed!}\;, $$ when the integrand is fixed because the path is fixed. (Note that the functional form of $L$ is always fixed for a given problem.)

But $$ \frac{\partial S_{cl}}{\partial t_1} = H(t_1) \tag{$L(x(t),\dot x(t))$ not fixed}\;, $$ when the integrand changes due to the change in classical path required for the different initial and final actions.

I will explain this in detail below.


I have two following questions mostly related to each other.

Q1: we ask how much S changes when we change $t_1$. This basically means we're not changing the path in variation, but we're just saying what if we had started the path from $t1+\delta h$ which means the remaining path is the same. Correct ?

Q2: I wonder how we derived $\frac{\partial S}{\partial t_1} = H(t1)$. My idea is as follows - we know that $H = vp - L$ from which we have: $L = vp - H$.

As an introduction, perhaps it is best to begin with an appeal to authority. And what greater authority than Dick Feynman, who posed the following question in his incomparable textbook "Quantum Mechanics and Path Integrals": Artist's rendition of Question 2.5 from Feynman and Hibbs

In his book, and in the above-quoted passage, Dick used the notation "$E$" for the Hamiltonian. However, we have been using the letter $E$ to denote the total mechanical energy (which is not always the same as the Hamiltonian, but often is). We have also been using $t_1$ for the initial time and $t_2$ for the final time rather than $t_a$ and $t_b$. So, for consistency, I have rewriten the relevant equations below using our notation: $$ H=\dot x p - L = \dot x \frac{\partial L}{\partial \dot x} - L $$ $$ H(t_2) = -\frac{\partial S_{cl}}{\partial t_2} $$ $$ H(t_1) = +\frac{\partial S_{cl}}{\partial t_2}\;. $$

Dick also provides us with a Hint: "A change in the time of an end point requires a change in path, since all paths must be classical paths."


As another introductory matter, consider the usual rule for differentiation with respect to an endpoint. If we define $$ F(t_1, t_2) = \int_{t_1}^{t_2}f(t')dt'\;, $$ then for fixed $f(t)$ (that is, for a fixed integrand function) we have: $$ \frac{\partial F}{\partial t_1} = -f(t_1) $$ and $$ \frac{\partial F}{\partial t_2} = +f(t_2)\;. $$

Thus, consistent with the hint we were given, the integrand (the Lagrangian evaluated on a specific classical path) must not be fixed or else we would simply arrive at: $$ \frac{\partial S_{cl}}{\partial t_1} \stackrel{??!}{=} -L(t_1)\tag{No, this is wrong!} $$ and $$ \frac{\partial S_{cl}}{\partial t_2} \stackrel{??!}{=} +L(t_2)\tag{No, this is wrong!} $$


To explain how I will proceed, another picture may be helpful: enter image description here

With reference to the above picture, we have:

  1. The classical path from $(t_1, x_1)$ to $(t_2, x_2)$ is $q(t)$
  2. The classical path from $(t_1+\delta t, x_1)$ to $(t_2, x_2)$ is r(t). (Note that $r(t)$ is also the path from $(t_1, x_1-\delta x)$ to $(t_2,x_2)$)
  3. The deviation from one path to the other is $\eta(t) = r(t)-q(t)$. This deviation will be "small" in the same sense that $\delta t$ and $\delta x$ are "small." That is, we will be content with only working to linear order in $\eta$ and $\delta t$ and $\delta x$ since we will take the limit $\delta t\to 0$ in the end.

Note that we have chosen $\eta(t)$ such that $\eta(t_1)=-\delta x$ and $\eta(t_2) = 0$. We made this choice because we are here considering only $\frac{\partial S_{cl}}{\partial t_1}$. (That is, here we are only going to show that $\frac{\partial S_{cl}}{\partial t_1}=H(t_1)$.)

The initial classical action of interest is: $$ S_{cl}^{\text{init}}=\int_{t_1}^{t_2}L(q(t'),\dot q(t'))dt'\;. $$

And the final classical action of interest is: $$ S_{cl}^{\text{final}}=\int_{t_1+\delta t}^{t_2}L(r(t'),\dot r(t'))dt'\;. $$

Thus, the relevant "small" change in the action $\delta S_{cl}$ that matters to us is: $$ \delta S_{cl} = \int_{t_1+\delta t}^{t_2}L(r(t'),\dot r(t'))dt' - \int_{t_1}^{t_2}L(q(t'),\dot q(t'))dt'\;, \tag{1}\;. $$

Continuing, we re-write our equation as: $$ \delta S_{cl} = \int_{t_1+\delta t}^{t_2}\left[L(r,\dot r) - L(q,\dot q)\right]dt' - \int_{t_1}^{t_1+\delta t}L(q,\dot q)dt' $$ $$ \approx \int_{t_1+\delta t}^{t_2}\left[L(r,\dot r) - L(r-\eta,\dot r-\dot \eta)\right]dt' -\delta t L(t_1)\;, $$ where we pause here to note that the last term above $-\delta t L(t_1)$ would be the only contribution to the change in the case that the integrand was fixed.

By expanding $L(q-\eta,\dot q - \dot \eta)$ to first order in $\eta$, we find: $$ \delta S_{cl}\approx \int_{t_1+\delta t}^{t_2}\left[\eta(t')\left.\frac{\partial L}{\partial x}\right|_{x=r}+\dot \eta \left.\frac{\partial L}{\partial \dot x}\right|_{x=r}\right]dt'-\delta t L(t_1)\;. $$ $$ =\int_{t_1+\delta t}^{t_2} \left[ \eta\frac{\partial L}{\partial x} +\frac{d}{dt}\left(\frac{\partial L}{\partial \dot x}\eta\right) -\eta\frac{d}{dt}\frac{\partial L}{\partial \dot x} \right]dt' -\delta t L(t_1) $$ $$ =\int_{t_1+\delta t}^{t_2} \left[\frac{d}{dt}\left(\frac{\partial L}{\partial \dot x}\eta\right) \right]dt' -\delta t L(t_1)\;, $$ where, in the last step, we used the Euler-Lagrange equation, since we are on the classical path from $t_1+\delta t$ to $t_2$.

Now, the integral in the first term above is trivial, since it is the integral of a total derivative. Performing the integration, and using $\eta(t_1+\delta t)\approx-\delta x$ and $\eta(t_2)=0$ we find $$ \delta S_{cl} \approx \delta x\frac{\partial L}{\partial \dot x}-\delta t L\;. $$

Or, taking the limit as $\delta t \to 0$ and ignoring terms of higher than linear order in $\delta t$, and using $\dot x=\lim_{\delta t\to 0}\frac{\delta x}{\delta t}$, we find $$ \frac{\partial S_{cl}}{\partial t_1}=\left. \dot{x} \frac{\partial L}{\partial \dot {x}}\right|_{x=q(t_1)} - L(t_1)=H(t_1)\;, $$ which is what we set out to prove.


The similar proof that $$ -\frac{\partial S_{cl}}{\partial t_2} = \dot x \frac{\partial L}{\partial \dot x}(t_2) - L(t_2) $$ is left as an exercise to the interested reader.


hft
  • 19,536
  • I will need to look at this in my own time but: 1. Are you missing a $\delta$ in the first term on the RHS in (1); 2. In going from (1) to (2), you dropped a $t_2$ and you interchanged some limits of integration without changing the sign…. – ZeroTheHero Jul 22 '23 at 04:19
  • Probably. It's late. I'll look it over again in the morning. But, you get the idea. – hft Jul 22 '23 at 04:55
  • @ZeroTheHero The entire calculation had an overall sign error (I was calculating the initial action minus the final action rather than vice versa). There were also two further sign errors in each term, which have now been corrected. I also added a section explaining why what we are doing here is different from the usual differentiation with respect to endpoints of an integral for a fixed integrand. Everything looks correct to me now... lots of opportunities for sign errors so it probably wasn't a good idea to try and write this up late last night ;) – hft Jul 22 '23 at 15:26
  • 1
    my confusion was that this includes variations in the endpoints. I went back to textbooks and resolved the issue quickly thereafter. I did not know this result (or at least didn’t remember it) so I learned something. – ZeroTheHero Jul 22 '23 at 15:45
  • 1
    Thanks so much for this. Today I had a crazy day and my brain cant comprehend anything. Will get back to this tomorrow morning. Thank you – Giorgi Jul 22 '23 at 17:24
  • Couple of questions. I'm almost there. 1) Why do you put ′ near t , such as ((′) ? I thought it's a derivative, but then i realized it's not. You even put it in 2) I think - (1+)≈− this is wrong, you should have (1+t)≈− 3. How do you make sure that approximation of (1+t)≈− won't take us into error ? i know (1) = −, but still. – Giorgi Jul 23 '23 at 21:38
  • I used $t'$ as an integration variable instead of $t$, because I didn't want to get the integration variable $t'$ and the $dt'$ indicating integration confused with $\delta t$; 2) Yes, that was a typo, I fixed it; 3) Because $eta$ is already "small" and so I assume the correction is higher order in the "small" quantities.
  • – hft Jul 23 '23 at 21:50
  • whats $eta$ and what do you mean by higher order ? we only have for which we substitute (t1+). 2. does this whole proof also prove why E = -dS/dt ? (note i don't use derivation with initial/final points. We could say H(t1) = H(t2) which is also E, so dS/dt1 = E and -dS/dt2 = E, but why would dS/dt would be E as well ?
  • – Giorgi Jul 23 '23 at 21:55
  • Sorry. When I wrote $eta$ I meant $\eta$, which is a Greek letter named "eta." – hft Jul 23 '23 at 21:57
  • still i got 2 questions remaining. 1. what do you mean by higher order ? we only have for which we substitute (t1+). 2. does this whole proof also prove why E = -dS/dt ? (note i don't use derivation with initial/final points. We could say H(t1) = H(t2) which is also E, so dS/dt1 = E and -dS/dt2 = E, but why would dS/dt would be E as well ? For 2nd question, maybe I should ask a new question ? – Giorgi Jul 23 '23 at 22:03
  • about 2nd question in my previous reply, thats the question I also asked here - https://physics.stackexchange.com/questions/773012/hamiltonian-energy-e-h-frac-partial-s-partial-t?noredirect=1&lq=1 - I didn't fully understand what QMechanic answered. You trully answer amazingly, so will wait for you ^_^ Infinite thanks. – Giorgi Jul 23 '23 at 22:19
  • @hft Beautiful answer! I know this post is somewhat old, but does "fixed" integrand mean one for which the classical path (and therefore the Lagrangian) is a known function? If the integrand is not fixed, does the Lagrangian depend parametrically on the endpoint time? – CW279 Feb 07 '24 at 18:45
  • @CW279 The functional form of the Lagrangian is fixed (for example, $L(q,v) = q^2 - v^2$ is the difference between two squares). What is not fixed is the classical path. The classical path (the one that minimizes the action) from $t_1$ to $t_2$ is generally different from the classical path from $t_3$ to $t_4$. In the derivation we set $t_3=t_1+\delta t$ and $t_4=t_2$ and there is still a difference in classical path since $t_1+\delta t\neq t_1$. The difference will be "small" since $t_1+\delta t$ is "close to" $t_1$. – hft Feb 07 '24 at 22:51
  • So, because the two classical paths are different (call them $x(t)$ and $\tilde x(t)$) the integrand is different. In the first case it is $L(x(t),\dot x(t))$ and in the second case it is $L(\tilde x(t), \dot{\tilde x}(t))$. – hft Feb 07 '24 at 22:53